The last outdoor

This topic has expert replies
User avatar
Master | Next Rank: 500 Posts
Posts: 117
Joined: Fri Jul 23, 2010 7:57 pm
Location: India
Thanked: 1 times

The last outdoor

by YellowSapphire » Thu Sep 30, 2010 7:47 am
Source: Veritas Prep CR2
102. The last outdoor drive-in movie theater in Nova Scotia closed recently. The owners claimed that it could not regularly attract large enough audience to remain viable. However, for an entire week of the theater's operation, after the announcement of the impending closure-the place was old out every night and made a healthy profit. Therefore, the owner's claim was clearly false.

Which one of the following contains an error of reasoning most similar to that made in the argument above?

A: On the many occasions similar to the present when the library's budget was cut, staff layoffs resulted, so even though the administration denies it, the proposed budget cuts are bound to mean staff layoffs.
B: The proposed cuts in library funding would require reducing the hours of the periodicals room. But that is a room in which many students like to study, so the proposed cuts are bound to mean staff layoffs.
C: All of the students who came to the meeting about proposed cuts in library funding were strongly opposed to the cuts, so all of the students currently enrolled must be strongly opposed.
D: The overall reduction in the university's budget is 10 percent. The library's budget is being cut by 10 percent. Therefore the library's budget cut is fair.
E: The administration claims that the library's funding must be cut because of last year's poor library fund drive. However the athletic department's budget is being increased. Therefore, the administration's claims must be false.

OA: C
Last edited by YellowSapphire on Fri Oct 01, 2010 7:16 pm, edited 1 time in total.
Yellow Sapphire

User avatar
Legendary Member
Posts: 1261
Joined: Sun Sep 14, 2008 3:46 am
Thanked: 27 times
GMAT Score:570

by reply2spg » Thu Sep 30, 2010 8:34 am
Is it C?
Sudhanshu
(have lot of things to learn from all of you)

User avatar
MBA Student
Posts: 113
Joined: Sat Dec 05, 2009 10:05 pm
Location: West Lafayette
Thanked: 1 times
GMAT Score:700

by g000fy » Thu Sep 30, 2010 10:43 am
I go with E

Senior | Next Rank: 100 Posts
Posts: 57
Joined: Thu Feb 26, 2009 3:25 pm
Thanked: 5 times
GMAT Score:730

by 007.r.mason » Thu Sep 30, 2010 2:01 pm
I see E as the closest match as well, although I don't like the choice. It is the best out of a set of bad choices!!
Aim high!!
Achieve excellence!!

Senior | Next Rank: 100 Posts
Posts: 70
Joined: Sat Aug 07, 2010 8:33 am

by SeemaSkl » Thu Sep 30, 2010 2:21 pm
I will go with C.

User avatar
Senior | Next Rank: 100 Posts
Posts: 74
Joined: Fri Jun 25, 2010 6:33 pm
Thanked: 3 times

by vijaynaik » Thu Sep 30, 2010 9:43 pm
IMO E

Senior | Next Rank: 100 Posts
Posts: 95
Joined: Wed Sep 15, 2010 10:16 pm

by psychomath » Fri Oct 01, 2010 4:59 am
Weird choices- E from the lot!

User avatar
Master | Next Rank: 500 Posts
Posts: 117
Joined: Fri Jul 23, 2010 7:57 pm
Location: India
Thanked: 1 times

by YellowSapphire » Fri Oct 01, 2010 8:38 am
Any expert reply please.
Yellow Sapphire

User avatar
Junior | Next Rank: 30 Posts
Posts: 15
Joined: Wed Jul 14, 2010 11:24 pm
Followed by:1 members

by samudranb » Fri Oct 01, 2010 8:48 am
I feel it is C
YellowSapphire wrote:Source: Veritas Prep CR2
101. The last outdoor drive-in movie theater in Nova Scotia closed recently. The owners claimed that it could not regularly attract large enough audience to remain viable. However, for an entire week of the theater's operation, after the announcement of the impending closure-the place was old out every night and made a healthy profit. Therefore, the owner's claim was clearly false.
The error in the reasoning can be described as something like this: "On announcement of an action, many people turned up against it. Hence, everybody must be opposed to it."

IMHO, C contains the same error of reasoning.
YellowSapphire wrote:

A: On the many occasions similar to the present when the library's budget was cut, staff layoffs resulted, so even though the administration denies it, the proposed budget cuts are bound to mean staff layoffs.
B: The proposed cuts in library funding would require reducing the hours of the periodicals room. But that is a room in which many students like to study, so the proposed cuts are bound to mean staff layoffs.
C: All of the students who came to the meeting about proposed cuts in library funding were strongly opposed to the cuts, so all of the students currently enrolled must be strongly opposed.
D: The overall reduction in the university's budget is 10 percent. The library's budget is being cut by 10 percent. Therefore the library's budget cut is fair.
E: The administration claims that the library's funding must be cut because of last year's poor library fund drive. However the athletic department's budget is being increased. Therefore, the administration's claims must be false.

User avatar
GMAT Instructor
Posts: 2193
Joined: Mon Feb 22, 2010 6:30 pm
Location: Vermont and Boston, MA
Thanked: 1186 times
Followed by:512 members
GMAT Score:770

by David@VeritasPrep » Fri Oct 01, 2010 6:56 pm
I believe this is question 102 in Veritas Prep CR2.

OA is C.

Here is a "mimic the reasoning" question. The correct answer will mimic the reasoning used in the stimulus. So start by understanding exactly what is happening in the stimulus.

In this stimulus you have a theater closing because of poor attendance, yet on the last week the theater was in operation, it made a healthy profit. The argument concludes that the owner's claim of poor attendance was false.

The flaw here is one of "over-generalization." The argument takes a single week of the theater's operation and concludes something about the overall attendance. This is like taking the score of a single student from the United States and concluding something about all students from the U.S.

And this flaw is even beyond that of simple over-generalization. In this case we have reason to believe that the particular week that is used as evidence is "atypical" of the theater attendance in general. Clearly, people might be there just to say goodbye. So what we have is over-generalization from evidence that is probably not typical.

The correct answer is going to be "C." In this case you have a sample of students that came to a meeting opposing cuts to the library and from that sample the answer concludes that all enrolled students are opposed to the cuts. So this is over-generalization. And in this case you also have reason to believe that this group is not typical of all students. Those most likely to show up at such a meeting are those who especially oppose the cuts to the library budget. So this matches very well the reasoning in the stimulus.

None of the other choices involve over-generalization.

Hope that helps! Any questions?
Veritas Prep | GMAT Instructor

Veritas Prep Reviews
Save $100 off any live Veritas Prep GMAT Course

GMAT/MBA Expert

User avatar
GMAT Instructor
Posts: 3380
Joined: Mon Mar 03, 2008 1:20 am
Thanked: 2256 times
Followed by:1535 members
GMAT Score:800

by lunarpower » Wed Oct 06, 2010 2:03 am
samudranb wrote:The error in the reasoning can be described as something like this: "On announcement of an action, many people turned up against it. Hence, everybody must be opposed to it."

IMHO, C contains the same error of reasoning.
very nice summary. concise, yet complete.

@veritas instructor: is there an official precedent for including this sort of problem in the CR section?
the only similar thing i can turn up, on brief inspection, is og12 *reading comp* problem 16 (on page 367 of og12). that's this sort of question ... but it isn't in CR.
so, i'm wondering which official problem was the precedent for this one.

(i know that these problems are all over the place on the LSAT cr section, but that's a totally different sort of exam)
Ron has been teaching various standardized tests for 20 years.

--

Pueden hacerle preguntas a Ron en castellano
Potete chiedere domande a Ron in italiano
On peut poser des questions à Ron en français
Voit esittää kysymyksiä Ron:lle myös suomeksi

--

Quand on se sent bien dans un vêtement, tout peut arriver. Un bon vêtement, c'est un passeport pour le bonheur.

Yves Saint-Laurent

--

Learn more about ron

User avatar
GMAT Instructor
Posts: 2193
Joined: Mon Feb 22, 2010 6:30 pm
Location: Vermont and Boston, MA
Thanked: 1186 times
Followed by:512 members
GMAT Score:770

by David@VeritasPrep » Wed Oct 06, 2010 6:57 am
Ron -

I am happy to help you with this since you ask. This is actually an official LSAT question. I have taught this one in numerous LSAT classes over the years. This is included in the "challenge" section of the Veritas CR2 book. Since about 1/3 of Veritas students score over 700 on the actual GMAT we just want them to have the option to be prepared for the tougher questions that they are likely to see at that level. It is true that many LSAT questions will only confuse students unnecessarily, but this is an excellent question for helping students to recognize the structure of an argument without relying on formal logic that is not needed on the GMAT.
Veritas Prep | GMAT Instructor

Veritas Prep Reviews
Save $100 off any live Veritas Prep GMAT Course

Legendary Member
Posts: 1119
Joined: Fri May 07, 2010 8:50 am
Thanked: 29 times
Followed by:3 members

by diebeatsthegmat » Wed Oct 06, 2010 2:47 pm
YellowSapphire wrote:Source: Veritas Prep CR2
102. The last outdoor drive-in movie theater in Nova Scotia closed recently. The owners claimed that it could not regularly attract large enough audience to remain viable. However, for an entire week of the theater's operation, after the announcement of the impending closure-the place was old out every night and made a healthy profit. Therefore, the owner's claim was clearly false.

Which one of the following contains an error of reasoning most similar to that made in the argument above?

A: On the many occasions similar to the present when the library's budget was cut, staff layoffs resulted, so even though the administration denies it, the proposed budget cuts are bound to mean staff layoffs.
B: The proposed cuts in library funding would require reducing the hours of the periodicals room. But that is a room in which many students like to study, so the proposed cuts are bound to mean staff layoffs.
C: All of the students who came to the meeting about proposed cuts in library funding were strongly opposed to the cuts, so all of the students currently enrolled must be strongly opposed.
D: The overall reduction in the university's budget is 10 percent. The library's budget is being cut by 10 percent. Therefore the library's budget cut is fair.
E: The administration claims that the library's funding must be cut because of last year's poor library fund drive. However the athletic department's budget is being increased. Therefore, the administration's claims must be false.

OA: C
i went with E and dunno why C. can you give an explaination?

Newbie | Next Rank: 10 Posts
Posts: 1
Joined: Sat Sep 25, 2010 3:14 am

by gmatjoyti » Thu Oct 07, 2010 2:43 am
^

Step 1: Breakdown to question type: Argument & Inference.

Step 2: restate: Statement: Owner states X ( losing money) because of y ( no drive in audience).
Argument: X proven invalid, because y increased in the last week
Inferred Premise: y during the last weeks is representative of all Y (Entire population)

Step 3: Elimination: A, B,D

Step 4: Possible choices C & E

C is the better choice because involves extrapolation of premise to entire population

E is also correct. Incorrect extrapolation of premise. But recognize the phrasing of Question stem: Which one of the following contains an error of reasoning most similar to that made in the argument above. C is the answer because it is "more" correct in identifying with the incorrect premise inferred in the passage